Varcarolis Ch 5 -9 Quiz

Lakukan tugas rumah & ujian kamu dengan baik sekarang menggunakan Quizwiz!

A nurse is interviewing a new client who is angry and highly suspicious. When asked about sexual orientation, the client becomes highly distressed and threatens to walk out of the interview. The nurse responds "I can see that this topic makes you uncomfortable. We can defer discussion of it today."

"I can see that this topic makes you uncomfortable. We can defer discussion of it today." Correct A cardinal rule of interviewing is "Don't probe sensitive areas." Clients are allowed to take the lead. REF: Page 117-118

You are interviewing Jamie, a 17-year-old female patient. She confides that she has been thinking of ways to kill a female peer who is Jamie's rival for the volleyball team captain position. She asks you if you can keep it a secret. The most appropriate response for you to make is: "Jamie, issues of this kind have to be shared with the treatment team and your parents."

"Jamie, issues of this kind have to be shared with the treatment team and your parents." Correct Although adolescent patients request confidentiality, issues of sexual abuse, threats of suicide or homicide, or issues that put the patient at risk for harm must be shared with the treatment team and the parents. A threat of this nature must be discussed with the treatment team and the parents. Confidentiality laws do not protect information that would lead to harm to the patient or others. This information would be shared with both the team and the parents

Which response to a patient's question of why you need to conduct an assessment interview best explains its purpose? "We will be able to form a relationship together where we can discuss the current problems and come up with goals and a plan for treatment."

"We will be able to form a relationship together where we can discuss the current problems and come up with goals and a plan for treatment." Correct Some of the purposes of the assessment interview are to establish rapport, learn more about the presenting issues, and form mutual goals and a plan for treatment. The other options do not appropriately explain the assessment purpose.

The nurse best assesses the client's spiritual life by asking, "What role does religion play in your life?"

"What role does religion play in your life?" Correct Asking the client to define the role of religion in their life allows for discussion related to the other topics. REF: Page 121-122

You are working in the emergency department when a 26-year-old male patient is brought in suffering from psychosis. The patient is unable to give any coherent history. The patient's best friend is with him and offers to give you information regarding the patient. Which of the following responses is appropriate? "Yes, I will be happy to get any information and history that you can provide."

"Yes, I will be happy to get any information and history that you can provide." Correct The friend is a secondary source of information that will be helpful since the patient is not able to give any history or information at this time. Confidentiality laws do not prohibit obtaining information from a secondary source. The friend can provide information and/or history immediately and may be able to relate events that happened just before coming to the hospital. A release would not be necessary to take information about the patient from a secondary source, and a psychotic patient would not be competent to sign a release.

Which viewpoint of an Asian American family will most affect decision making about care? a. The father is the authority figure. b. The mother is head of the household. c. Women should make their own decisions. d. Emotional communication styles are desirable.

A Asian American families traditionally place the father in the position of power as the head of the household. Mothers, as well as other women, are usually subservient to fathers in these cultures. Asian Americans are more likely to be reserved

An experienced psychiatric nurse plans to begin a new job in a community-based medication clinic. The clinic sees culturally diverse patients. Which action should the nurse take first to prepare for this position? a. Investigate cultural differences in patients' responses to psychotropic medications. b. Contact the clinical nurse specialist for guidelines regarding cultural competence. c. Examine the literature on various health beliefs of members of diverse cultures. d. Complete an online continuing education offering about psychopharmacology.

A An experienced nurse working on a mental health inpatient unit would be familiar with the action and side effects of most commonly prescribed psychotropic medications. However, because the clinic serves a culturally diverse population, reviewing cultural differences in patients' responses to these medications is helpful and vital to patient safety. The distracters identify actions the nurse would take later.

The sibling of an Asian American patient tells the nurse, "My sister needs help for pain. She cries from the hurt." Which understanding by the nurse will contribute to culturally competent care for this patient? Persons of an Asian American heritage a. often express emotional distress with physical symptoms. b. will probably respond best to a therapist who is impersonal. c. will require prolonged treatment to stabilize these symptoms. d. should be given direct information about the diagnosis and prognosis

A Asian Americans commonly express psychological distress as a physical problem. The patient may believe psychological problems are caused by a physical imbalance. Treatment will likely be short. The patient will probably respond best to a therapist who is perceived as giving. Asian Americans usually have strong family ties and value hope more than truth.

Which action by a psychiatric nurse best applies the ethical principle of autonomy? a. Exploring alternative solutions with the patient, who then makes a choice. b. Suggesting that two patients who were fighting be restricted to the unit. c. Intervening when a self-mutilating patient attempts to harm self. d. Staying with a patient demonstrating a high level of anxiety.

A Autonomy is the right to self-determination, that is, to make one's own decisions. By exploring alternatives with the patient, the patient is better equipped to make an informed, autonomous decision. The distracters demonstrate beneficence, fidelity, and justice.

A nurse speaks with family members of a Chinese American parent recently diagnosed with major depressive disorder. Which comment by the nurse will the family find most comforting? "The nursing staff will a. take good care of your parent." b. pray with your parent several times a day." c. teach your parent important self-care strategies." d. educate your parent about safety information regarding medication."

A Chinese Americans hold an Eastern (balance) worldview. Persons who are ill or need health care are vulnerable and need protection. The family will find comfort in a nurse's statement that good care will be provided. The distracters apply to persons with a Western or indigenous worldview.

The nurse administers medications to a culturally diverse group of patients on a psychiatric unit. What expectation should the nurse have about pharmacokinetics? a. Patients of different cultural groups may metabolize medications at different rates. b. Metabolism of psychotropic medication is consistent among various cultural groups. c. Differences in hepatic enzymes will influence the rate of elimination of psychotropic medications. d. It is important to provide patients with oral and written literature about their psychotropic medications.

A Cytochrome enzyme systems, which vary among different cultural groups, influence the rate of metabolism of psychoactive drugs. Renal function influences elimination of psychotropic medication: hepatic function influences metabolism rates.

While talking with a patient diagnosed with major depressive disorder, a nurse notices the patient is unable to maintain eye contact. The patient's chin lowers to the chest. The patient looks at the floor. Which aspect of communication has the nurse assessed? a. Nonverbal communication b. A message filter c. A cultural barrier d. Social skills

A Eye contact and body movements are considered nonverbal communication. There are insufficient data to determine the level of the patient's social skills or an existing cultural barrier.

A patient is having difficulty making a decision. The nurse has mixed feelings about whether to provide advice. Which principle usually applies? Giving advice a. is rarely helpful. b. fosters independence. c. lifts the burden of personal decision making. d. helps the patient develop feelings of personal adequacy.

A Giving advice fosters dependence on the nurse and interferes with a patient's right to make personal decisions. It robs the patient of the opportunity to weigh alternatives and develop problem-solving skills. Furthermore, it may contribute to a patient's feelings of personal inadequacy. Giving advice also keeps the nurse in control and feeling powerful.

A nurse in the clinic has a full appointment schedule. A Hispanic American patient arrives at 1230 for a 1000 appointment. A Native American patient does not keep an appointment at all. What understanding will improve the nurse's planning? These patients are a. members of cultural groups that have a different view of time. b. immature and irresponsible in health care matters. c. acting-out feelings of anger toward the system. d. displaying passive-aggressive tendencies.

A Hispanic Americans and Native Americans traditionally treat time in a way unlike the Western culture. They tend to be present-oriented- that is, they value the current interaction more than what is to be done in the future. If engaged in an activity, for example, they may simply continue the activity and appear later for an appointment. Understanding this, the nurse can avoid feelings of frustration and anger when the nurse's future orientation comes into conflict with the patient's present orientation.

A psychiatric nurse leads a medication education group for Hispanic patients. This nurse holds a Western worldview and uses pamphlets as teaching tools. Groups are short and concise. After the group, the patients are most likely to believe a. the nurse was uncaring. b. the session was effective. c. the teaching was efficient. d. they were treated respectfully.

A Hispanic individuals usually value relationship behaviors. Their needs are for learning through verbal communication rather than reading and for having time to chat before approaching the task.

A nurse prepares to teach important medication information to a patient of Mexican heritage. How should the nurse manage the teaching environment? a. Stand very close to the patient while teaching. b. Maintain direct eye contact with the patient while teaching. c. Maintain a neutral emotional tone during the teaching session. d. Sit 4 feet or more from the patient during the teaching session.

A Latin American cultures use close personal space, closer than many other minority groups. Standing very close to the patient frequently indicates acceptance. Direct eye contact should not be prolonged with this patient. Persons of this cultural heritage have high emotionality.

A patient in alcohol rehabilitation reveals to the nurse, "I feel terrible guilt for sexually abusing my 6-year-old before I was admitted." Select the nurse's most important action. a. Anonymously report the abuse by phone to the local child protection agency. b. Reply, "I'm glad you feel comfortable talking to me about it." c. File a written report with the agency's ethics committee. d. Respect nurse-patient relationship confidentiality.

A Laws regarding child abuse reporting discovered by a professional during the suspected abuser's alcohol or drug treatment differ by state. Federal law supersedes state law and prohibits disclosure without a court order except in instances in which the report can be made anonymously or without identifying the abuser as a patient in an alcohol or drug treatment facility.

During a one-on-one interaction with the nurse, a patient frequently looks nervously at the door. Select the best comment by the nurse regarding this nonverbal communication. a. "I notice you keep looking toward the door." b. "This is our time together. No one is going to interrupt us." c. "It looks as if you are eager to end our discussion for today." d. "If you are uncomfortable in this room, we can move someplace else."

A Making observations and encouraging the patient to describe perceptions are useful therapeutic communication techniques for this situation. The other responses are assumptions made by the nurse.

A nurse is concerned that an agency's policies are inadequate. Which understanding about the relationship between substandard institutional policies and individual nursing practice should guide nursing practice? a. Agency policies do not exempt an individual nurse of responsibility to practice according to professional standards of nursing care. b. Agency policies are the legal standard by which a professional nurse must act and therefore override other standards of care. c. Faced with substandard policies, a nurse has a responsibility to inform the supervisor and discontinue patient care immediately. d. Interpretation of policies by the judicial system is rendered on an individual basis and therefore cannot be predicted.

A Nurses are professionally bound to uphold standards of practice regardless of lesser standards established by a health care agency or a state. Conversely, if the agency standards are higher than standards of practice, the agency standards must be upheld. The Courts may seek to establish the standard of care through the use of expert witnesses when the issue is clouded.

In order to release information to another health care facility or third party regarding a patient diagnosed with a mental illness, the nurse must obtain a. a signed consent by the patient for release of information stating specific information to be released. b. a verbal consent for information release from the patient and the patient's guardian or next of kin. c. permission from members of the health care team who participate in treatment planning. d. approval from the attending psychiatrist to authorize the release of information.

A Nurses have an obligation to protect patients' privacy and confidentiality. Clinical information should not be released without the patient's signed consent for the release.

Which technique will best communicate to a patient that the nurse is interested in listening? a. Restating a feeling or thought the patient has expressed. b. Asking a direct question, such as "Did you feel angry?" c. Making a judgment about the patient's problem. d. Saying, "I understand what you're saying."

A Restating allows the patient to validate the nurse's understanding of what has been communicated. Restating is an active listening technique. Judgments should be suspended in a nurse-patient relationship. Close-ended questions such as "Did you feel angry?" ask for specific information rather than showing understanding. When the nurse simply states that he or she understands the patient's words, the patient has no way of measuring the understanding.

A person in the community asks, "Why aren't people with mental illness kept in state institutions anymore?" Select the nurse's best response. a. "Less restrictive settings are available now to care for individuals with mental illness." b. "There are fewer persons with mental illness, so less hospital beds are needed." c. "Most people with mental illness are still in psychiatric institutions." d. "Psychiatric institutions violated patients' rights."

A The community is a less restrictive alternative than hospitals for treatment of persons with mental illness. The distracters are incorrect and part of the stigma of mental illness.

A Chinese American patient diagnosed with an anxiety disorder says, "My problems began when my energy became imbalanced." The nurse asks for the patient's ideas about how to treat the imbalance. Which comment would the nurse expect from this patient? a. "My family will bring special foods to help me get well." b. "I hope my health care provider will prescribe some medication to help me." c. "I think I would benefit from talking to other patients with a similar problem." d. "I would like to have a native healer perform a ceremony to balance my energy."

A The concept of energy imbalance as a source of illness is an explanatory model familiar to Asian cultures.

When a female Mexican American patient and a female nurse sit together, the patient often holds the nurse's hand. The patient also links arms with the nurse when they walk. The nurse is uncomfortable with this behavior. Which analysis is most accurate? a. The patient is accustomed to touch during conversation, as are members of many Hispanic subcultures. b. The patient understands that touch makes the nurse uncomfortable and controls the relationship based on that factor. c. The patient is afraid of being alone. When touching the nurse, the patient is reassured and comforted. d. The patient is trying to manipulate the nurse using nonverbal techniques.

A The most likely answer is that the patient's behavior is culturally influenced. Hispanic women frequently touch women they consider to be their friends. Although the other options are possible, they are less likely.

A patient with acute depression states, "God is punishing me for my past sins." What is the nurse's most therapeutic response? a. "You sound very upset about this." b. "God always forgives us for our sins." c. "Why do you think you are being punished?" d. "If you feel this way, you should talk to your minister."

A The nurse reflects the patient's comment, a therapeutic technique to encourage sharing for perceptions and feelings. The incorrect responses reflect probing, closed-ended comments, and giving advice, all of which are nontherapeutic.

Insurance will not pay for continued private hospitalization of a mentally ill patient. The family considers transferring the patient to a public hospital but expresses concern that the patient will not get any treatment if transferred. Select the nurse's most helpful reply. a. "By law, treatment must be provided. Hospitalization without treatment violates patients' rights." b. "All patients in public hospitals have the right to choose both a primary therapist and a primary nurse." c. "You have a justifiable concern because the right to treatment extends only to provision of food, shelter, and safety." d. "Much will depend on other patients, because the right to treatment for a psychotic patient takes precedence over the right to treatment of a patient who is stable."

A The right to medical and psychiatric treatment is conferred on all patients hospitalized in public mental hospitals under federal law.

To provide culturally competent care, the nurse should a. accurately interpret the thinking of individual patients. b. predict how a patient may perceive treatment interventions. c. formulate interventions to reduce the patient's ethnocentrism. d. identify strategies that fit within the cultural context of the patient.

D The correct answer is the most global response.

Which benefits are most associated with use of telehealth technologies? (Select all that apply.) a. Cost savings for patients b. Maximize care management c. Access to services for patients in rural areas d. Prompt reimbursement by third-party payers e. Rapid development of trusting relationships with patients

A, B, C Telehealth has shown that it can maximize health and improve disease management skills and confidence with the disease process. Many rural parents have felt disconnected from services- telehealth technologies can solve those problems. Although telehealth's improved health outcomes regularly show cost savings for payers, one significant barrier is the current lack of reimbursement for remote patient monitoring by third-party payers. Telehealth technologies have not shown rapid development of trusting relationships.

The nurse should be particularly alert to expression of psychological distress through physical symptoms among patients whose cultural beliefs include (Select all that apply) a. mental illness reflects badly on the family. b. mental illness shows moral weakness. c. intergenerational conflict is common. d. the mind, body, and spirit are merged. e. food choices influence one's health.

A, B, D Physical symptoms are seen as more acceptable in cultural groups in which interdependence and harmony of the group are emphasized. Mental illness is often perceived as reflecting a failure of the entire family. In groups in which mental illness is seen as a moral weakness and both the individual and family are stigmatized, somatization of mental distress is better accepted. In groups in which mind, body, and spirit are holistically perceived, somatization of psychological distress is common. Somatization and food are not commonly related. Intergenerational conflict has not been noted as a risk factor for somatization.

Which questions should the nurse ask to determine an individual's worldview? (Select all that apply.) a. What is more important: the needs of an individual or the needs of a community? b. How would you describe an ideal relationship between individuals? c. How long have you lived at your present residence? d. Of what importance are possessions in your life? e. Do you speak any foreign languages?

A, B, D The answers provide information about cultural values related to the importance of individuality, material possessions, relational connectedness, community needs versus individual needs, and interconnectedness between humans and nature. These will assist the nurse to determine a patient's worldview.

A nurse interacts with patients diagnosed with various mental illnesses. Which statements reflect use of therapeutic communication? (Select all that apply.) a. "Tell me more about that situation." b. "Let's talk about something else." c. "I notice you are pacing a lot." d. "I'll stay with you a while." e. "Why did you do that?"

A, C, D The correct responses demonstrate use of the therapeutic techniques making an observation and showing empathy. The incorrect responses demonstrate changing the subject and probing, which are nontherapeutic techniques

. During a mental status examination, a client who is hospitalized states that she is in the hospital "to help out with the other patients." The nurse should record this information as A. poor insight.

A. poor insight. The nurse's objective assessment of the client's insight reflects the client's understanding of her current situation and medical condition. Knowledge, judgment, and memory are other objective cognitive assessments. None reflect the client's understanding of the responsibility for, or analysis of, the current situation.

4. A patient says, "Please don't share information about me with the other people." How should the nurse respond? a. "I will not share information with your family or friends without your permission, but I will share information about you with other staff." b. "A therapeutic relationship is just between the nurse and the patient. It is up to you to tell others what you want them to know." c. "It depends on what you choose to tell me. I will be glad to disclose at the end of each session what I will report to others." d. "I cannot tell anyone about you. It will be as though I am talking about my own problems, and we can help each other by keeping it between us."

ANS: A A patient has the right to know with whom the nurse will share information and that confidentiality will be protected. Although the relationship is primarily between the nurse and patient, other staff needs to know pertinent data. The other incorrect responses promote incomplete disclosure on the part of the patient, require daily renegotiation of an issue that should be resolved as the nurse-patient contract is established, and suggest mutual problem solving. The relationship must be patient centered. See relationship to audience response question.

24. A community mental health nurse has worked with a patient for 3 years but is moving out of the city and terminates the relationship. When a novice nurse begins work with this patient, what is the starting point for the relationship? a. Begin at the orientation phase. b. Resume the working relationship. c. Initially establish a social relationship. d. Return to the emotional catharsis phase.

ANS: A After termination of a long-term relationship, the patient and new nurse usually have to begin at ground zero, the orientation phase, to build a new relationship. If termination is successfully completed, the orientation phase sometimes progresses quickly to the working phase. Other times, even after successful termination, the orientation phase may be prolonged.

17. A patient says, "I've done a lot of cheating and manipulating in my relationships." Select a nonjudgmental response by the nurse. a. "How do you feel about that?" b. "I am glad that you realize this." c. "That's not a good way to behave." d. "Have you outgrown that type of behavior?"

ANS: A Asking a patient to reflect on feelings about his or her actions does not imply any judgment about those actions, and it encourages the patient to explore feelings and values. The remaining options offer negative judgments.

1. A nurse assesses a confused older adult. The nurse experiences sadness and reflects, "This patient is like one of my grandparents ... so helpless." Which response is the nurse demonstrating? a. Transference b. Countertransference c. Catastrophic reaction d. Defensive coping reaction

ANS: B Countertransference is the nurse's transference or response to a patient that is based on the nurse's unconscious needs, conflicts, problems, or view of the world. See relationship to audience response question.

2. Which statement shows a nurse has empathy for a patient who made a suicide attempt? a. "You must have been very upset when you tried to hurt yourself." b. "It makes me sad to see you going through such a difficult experience." c. "If you tell me what is troubling you, I can help you solve your problems." d. "Suicide is a drastic solution to a problem that may not be such a serious matter."

ANS: A Empathy permits the nurse to see an event from the patient's perspective, understand the patient's feelings, and communicate this to the patient. The incorrect responses are nurse-centered (focusing on the nurse's feelings rather than the patient's), belittling, and sympathetic.

13. A nurse explains to the family of a mentally ill patient how a nurse-patient relationship differs from social relationships. Which is the best explanation? a. "The focus is on the patient. Problems are discussed by the nurse and patient, but solutions are implemented by the patient." b. "The focus shifts from nurse to patient as the relationship develops. Advice is given by both, and solutions are implemented." c. "The focus of the relationship is socialization. Mutual needs are met, and feelings are shared openly." d. "The focus is creation of a partnership in which each member is concerned with growth and satisfaction of the other."

ANS: A Only the correct response describes elements of a therapeutic relationship. The remaining responses describe events that occur in social or intimate relationships

22. As a nurse discharges a patient, the patient gives the nurse a card of appreciation made in an arts and crafts group. What is the nurse's best action? a. Recognize the effectiveness of the relationship and patient's thoughtfulness. Accept the card. b. Inform the patient that accepting gifts violates policies of the facility. Decline the card. c. Acknowledge the patient's transition through the termination phase but decline the card. d. Accept the card and invite the patient to return to participate in other arts and crafts groups.

ANS: A The nurse must consider the meaning, timing, and value of the gift. In this instance, the nurse should accept the patient's expression of gratitude. See relationship to audience response question.

9. At what point in the nurse-patient relationship should a nurse plan to first address termination? a. During the orientation phase b. At the end of the working phase c. Near the beginning of the termination phase d. When the patient initially brings up the topic

ANS: A The patient has a right to know the conditions of the nurse-patient relationship. If the relationship is to be time-limited, the patient should be informed of the number of sessions. If it is open-ended, the termination date will not be known at the outset, and the patient should know that the issue will be negotiated at a later date. The nurse is responsible for bringing up the topic of termination early in the relationship, usually during the orientation phase.

26. A nurse says, "I am the only one who truly understands this patient. Other staff members are too critical." The nurse's statement indicates a. boundary blurring. b. sexual harassment. c. positive regard. d. advocacy.

ANS: A When the role of the nurse and the role of the patient shift, boundary blurring may arise. In this situation the nurse is becoming overinvolved with the patient as a probable result of unrecognized countertransference. When boundary issues occur, the need for supervision exists. The situation does not describe sexual harassment. Data are not present to suggest positive regard or advocacy.

1. A nurse ends a relationship with a patient. Which actions by the nurse should be included in the termination phase? (Select all that apply.) a. Focus dialogues with the patient on problems that may occur in the future. b. Help the patient express feelings about the relationship with the nurse. c. Help the patient prioritize and modify socially unacceptable behaviors. d. Reinforce expectations regarding the parameters of the relationship. e. Help the patient to identify strengths, limitations, and problems.

ANS: A, B The correct actions are part of the termination phase. The other actions would be used in the working and orientation phases.

2. A novice psychiatric nurse has a parent diagnosed with bipolar disorder. This nurse angrily recalls feelings of embarrassment about the parent's behavior in the community. Select the best ways for this nurse to cope with these feelings. (Select all that apply.) a. Seek ways to use the understanding gained from childhood to help patients cope with their own illnesses. b. Recognize that these feelings are unhealthy. The nurse should try to suppress them when working with patients. c. Recognize that psychiatric nursing is not an appropriate career choice. Explore other nursing specialties. d. The nurse should begin new patient relationships by saying, "My own parent had mental illness, so I accept it without stigma." e. Recognize that the feelings may add sensitivity to the nurse's practice, but supervision is important.

ANS: A, E The nurse needs support to explore these feelings. An experienced psychiatric nurse is a resource that may be helpful. The knowledge and experience gained from the nurse's relationship with a mentally ill parent may contribute sensitivity to compassionate practice. Self-disclosure and suppression are not adaptive coping strategies. The nurse should not give up on this area of practice without first seeking ways to cope with the memories.

10. A nurse introduces the matter of a contract during the first session with a new patient because contracts a. specify what the nurse will do for the patient. b. spell out the participation and responsibilities of each party. c. indicate the feeling tone established between the participants. d. are binding and prevent either party from prematurely ending the relationship.

ANS: B A contract emphasizes that the nurse works with the patient rather than doing something for the patient. "Working with" is a process that suggests each party is expected to participate and share responsibility for outcomes. Contracts do not, however, stipulate roles or feeling tone, and premature termination is forbidden.

20. An advanced practice nurse observes a novice nurse expressing irritability regarding a patient with a long history of alcoholism and suspects the new nurse is experiencing countertransference. Which comment by the new nurse confirms this suspicion? a. "This patient continues to deny problems resulting from drinking." b. "My parents were alcoholics and often neglected our family." c. "The patient cannot identify any goals for improvement." d. "The patient said I have many traits like her mother."

ANS: B Countertransference occurs when the nurse unconsciously and inappropriately displaces onto the patient feelings and behaviors related to significant figures in the nurse's past. In this instance, the new nurse's irritability stems from relationships with parents. The distracters indicate transference or accurate analysis of the patient's behavior.

14. A nurse wants to demonstrate genuineness with a patient diagnosed with schizophrenia. The nurse should a. restate what the patient says. b. use congruent communication strategies. c. use self-revelation in patient interactions. d. consistently interpret the patient's behaviors.

ANS: B Genuineness is a desirable characteristic involving awareness of one's own feelings as they arise and the ability to communicate them when appropriate. The incorrect options are undesirable in a therapeutic relationship.

16. A nurse wants to enhance growth of a patient by showing positive regard. The nurse's action most likely to achieve this goal is a. making rounds daily. b. staying with a tearful patient. c. administering medication as prescribed. d. examining personal feelings about a patient.

ANS: B Staying with a crying patient offers support and shows positive regard. Administering daily medication and making rounds are tasks that could be part of an assignment and do not necessarily reflect positive regard. Examining feelings regarding a patient addresses the nurse's ability to be therapeutic.

25. As a patient diagnosed with a mental illness is being discharged from a facility, a nurse invites the patient to the annual staff picnic. What is the best analysis of this scenario? a. The invitation facilitates dependency on the nurse. b. The nurse's action blurs the boundaries of the therapeutic relationship. c. The invitation is therapeutic for the patient's diversional activity deficit. d. The nurse's action assists the patient's integration into community living.

ANS: B The invitation creates a social relationship rather than a therapeutic relationship.

3. A novice nurse tells a mentor, "I want to convey to my patients that I am interested in them and that I want to listen to what they have to say." Which behaviors will be helpful in meeting the nurse's goal? (Select all that apply.) a. Sitting behind a desk, facing the patient b. Introducing self to a patient and identifying own role c. Maintaining control of discussions by asking direct questions d. Using facial expressions to convey interest and encouragement e. Assuming an open body posture and sometimes mirror imaging

ANS: B, D, E Trust is fostered when the nurse gives an introduction and identifies his or her role. Facial expressions that convey interest and encouragement support the nurse's verbal statements to that effect and strengthen the message. An open body posture conveys openness to listening to what the patient has to say. Mirror imaging enhances patient comfort. A desk would place a physical barrier between the nurse and patient. A face-to-face stance should be avoided when possible and a less intense 90- or 120-degree angle used to permit either party to look away without discomfort.

21. Which behavior shows that a nurse values autonomy? The nurse a. suggests one-on-one supervision for a patient who has suicidal thoughts. b. informs a patient that the spouse will not be in during visiting hours. c. discusses options and helps the patient weigh the consequences. d. sets limits on a patient's romantic overtures toward the nurse.

ANS: C A high level of valuing is acting on one's belief. Autonomy is supported when the nurse helps a patient weigh alternatives and their consequences before the patient makes a decision. Autonomy or self-determination is not the issue in any of the other behaviors.

11. As a nurse escorts a patient being discharged after treatment for major depression, the patient gives the nurse a necklace with a heart pendant and says, "Thank you for helping mend my broken heart." Which is the nurse's best response? a. "Accepting gifts violates the policies and procedures of the facility." b. "I'm glad you feel so much better now. Thank you for the beautiful necklace." c. "I'm glad I could help you, but I can't accept the gift. My reward is seeing you with a renewed sense of hope." d. "Helping people is what nursing is all about. It's rewarding to me when patients recognize how hard we work."

ANS: C Accepting a gift creates a social rather than therapeutic relationship with the patient and blurs the boundaries of the relationship. A caring nurse will acknowledge the patient's gesture of appreciation, but the gift should not be accepted. See relationship to audience response question.

27. Which comment best indicates that a patient perceived the nurse was caring? "My nurse a. always asks me which type of juice I want to help me swallow my medication." b. explained my treatment plan to me and asked for my ideas about how to make it better." c. spends time listening to me talk about my problems. That helps me feel like I am not alone." d. told me that if I take all the medicines the doctor prescribes, then I will get discharged sooner."

ANS: C Caring evidences empathetic understanding as well as competency. It helps change pain and suffering into a shared experience, creating a human connection that alleviates feelings of isolation. The distracters give examples of statements that demonstrate advocacy or giving advice.

7. What is the desirable outcome for the orientation stage of a nurse-patient relationship? The patient will demonstrate behaviors that indicate a. self-responsibility and autonomy. b. a greater sense of independence. c. rapport and trust with the nurse. d. resolved transference.

ANS: C Development of rapport and trust is necessary before the relationship can progress to the working phase. Behaviors indicating a greater sense of independence, self-responsibility, and resolved transference occur in the working phase.

8. During which phase of the nurse-patient relationship can the nurse anticipate that identified patient issues will be explored and resolved? a. Preorientation b. Orientation c. Working d. Termination

ANS: C During the working phase, the nurse strives to assist the patient in making connections among dysfunctional behaviors, thinking, and emotions and offers support while alternative coping behaviors are tried.

23. A patient says, "I'm still on restriction, but I want to attend some off-unit activities. Would you ask the doctor to change my privileges?" What is the nurse's best response? a. "Why are you asking me when you're able to speak for yourself?" b. "I will be glad to address it when I see your doctor later today." c. "That's a good topic for you to discuss with your doctor." d. "Do you think you can't speak to a doctor?"

ANS: C Nurses should encourage patients to work at their optimal level of functioning, which in turn promotes autonomy. A nurse does not act for the patient unless it is necessary. Acting for a patient increases feelings of helplessness and dependency.

19. Which issues should a nurse address during the first interview with a patient with a psychiatric disorder? a. Trust, congruence, attitudes, and boundaries b. Goals, resistance, unconscious motivations, and diversion c. Relationship parameters, the contract, confidentiality, and termination d. Transference, countertransference, intimacy, and developing resources

ANS: C Relationship parameters, the contract, confidentiality, and termination are issues that should be considered during the orientation phase of the relationship. The remaining options are issues that are dealt with later.

3. After several therapeutic encounters with a patient who recently attempted suicide, which occurrence should cause the nurse to consider the possibility of countertransference? a. The patient's reactions toward the nurse seem realistic and appropriate. b. The patient states, "Talking to you feels like talking to my parents." c. The nurse feels unusually happy when the patient's mood begins to lift. d. The nurse develops a trusting relationship with the patient.

ANS: C Strong positive or negative reactions toward a patient or over-identification with the patient indicate possible countertransference. Nurses must carefully monitor their own feelings and reactions to detect countertransference and then seek supervision. Realistic and appropriate reactions from a patient toward a nurse are desirable. One incorrect response suggests transference. A trusting relationship with the patient is desirable. See relationship to audience response question.

6. Termination of a therapeutic nurse-patient relationship has been successful when the nurse a. avoids upsetting the patient by shifting focus to other patients before the discharge. b. gives the patient a personal telephone number and permission to call after discharge. c. discusses with the patient changes that happened during the relationship and evaluates outcomes. d. offers to meet the patient for coffee and conversation three times a week after discharge.

ANS: C Summarizing and evaluating progress help validate the experience for the patient and the nurse and facilitate closure. Termination must be discussed; avoiding discussion by spending little time with the patient promotes feelings of abandonment. Successful termination requires that the relationship be brought to closure without the possibility of dependency-producing ongoing contact.

15. A nurse caring for a withdrawn, suspicious patient recognizes development of feelings of anger toward the patient. The nurse should a. suppress the angry feelings. b. express the anger openly and directly with the patient. c. tell the nurse manager to assign the patient to another nurse. d. discuss the anger with a clinician during a supervisory session.

ANS: D The nurse is accountable for the relationship. Objectivity is threatened by strong positive or negative feelings toward a patient. Supervision is necessary to work through countertransference feelings.

12. Which remark by a patient indicates passage from orientation to the working phase of a nurse-patient relationship? a. "I don't have any problems." b. "It is so difficult for me to talk about problems." c. "I don't know how it will help to talk to you about my problems." d. "I want to find a way to deal with my anger without becoming violent."

ANS: D Thinking about a more constructive approach to dealing with anger indicates a readiness to make a behavioral change. Behavioral change is associated with the working phase of the relationship. Denial is often seen in the orientation phase. It is common early in the relationship, before rapport and trust are firmly established, for a patient to express difficulty in talking about problems. Stating skepticism about the effectiveness of the nurse-patient relationship is more typically a reaction during the orientation phase.

18. A patient says, "People should be allowed to commit suicide without interference from others." A nurse replies, "You're wrong. Nothing is bad enough to justify death." What is the best analysis of this interchange? a. The patient is correct. b. The nurse is correct. c. Neither person is correct. d. Differing values are reflected in the two statements.

ANS: D Values guide beliefs and actions. The individuals stating their positions place different values on life and autonomy. Nurses must be aware of their own values and be sensitive to the values of others.

5. A nurse is talking with a patient, and 5 minutes remain in the session. The patient has been silent most of the session. Another patient comes to the door of the room, interrupts, and says to the nurse, "I really need to talk to you." The nurse should a. invite the interrupting patient to join in the session with the current patient. b. say to the interrupting patient, "I am not available to talk with you at the present time." c. end the unproductive session with the current patient and spend time with the interrupting patient. d. tell the interrupting patient, "This session is 5 more minutes; then I will talk with you."

ANS: D When a specific duration for sessions has been set, the nurse must adhere to the schedule. Leaving the first patient would be equivalent to abandonment and would destroy any trust the patient had in the nurse. Adhering to the contract demonstrates that the nurse can be trusted and that the patient and the sessions are important. The incorrect responses preserve the nurse-patient relationship with the silent patient but may seem abrupt to the interrupting patient, abandon the silent patient, or fail to observe the contract with the silent patient.

When interviewing an adolescent client, the nurse can expect the client to be most concerned about the issue of confidentiality.

confidentiality. Correct Adolescents are often concerned that what they reveal to the nurse or health care team will be shared with parents. Confidentiality should be explained at the outset of the interview.REF: Page 118

What is the common behavior shared by both client and nurse at the beginning of the initial assessment interview? Anxiety

Anxiety Correct Both parties feel at least a small amount of anxiety associated with interacting with an unknown person. REF: Page 117-118

A psychoeducational session will discuss medication management for a culturally diverse group of patients. Group participants are predominantly members of minority cultures. Of the four staff nurses below, which nurse should lead this group? a. Very young registered nurse b. Older, mature registered nurse c. Newly licensed registered nurse d. A registered nurse who is very thin

B Persons of minority cultures value age and wisdom. Persons with a Western worldview tend to value youth. An older, mature registered nurse would be the most credible leader of this group. The nurse's size has no bearing on credibility.

A client is admitted to an acute care mental health facility. The following medical diagnoses and psychosocial information are available at the time of admission: hypothyroidism, mild mental retardation, bipolar I disorder. The client's highest level of functioning from a global assessment of functioning (GAF) performed a year ago was 45. Today, the highest level of functioning on the same scale is 15. The client has been fighting with other clients frequently at the group home. How should the nurse enter this information into the multiaxial system of the DSM- IV- TR? Axis I: Bipolar I disorder Axis II: Mild mental retardation Axis III: Hypothyroidism Axis IV: Has been fighting with other clients frequently at group home Axis V: GAF 15/45

Axis I includes most mental health clinical disorders, except those placed on Axis II. Axis II disorders include personality disorders and mental retardation. Axis III includes general medical disorders and problems. Axis IV includes pertinent psychosocial information or problems with living conditions. Axis V includes GAF for present assessment and previous assessment within 1 year of present.

A nurse wants to engage an interpreter for a severely anxious 21-year-old male who immigrated to the United States 2 years ago. Of the four interpreters below who are available and fluent in the patient's language, which one should the nurse call? a. 65-year-old female professional interpreter b. 24-year-old male professional interpreter c. A member of the patient's family d. The patient's best friend

B A professional interpreter will be most effective because he/she will be able to interpret both language and culture. When an interpreter is engaged, the interpreter should be matched to the patient as closely as possible in gender, age, social status, and religion. Interpreters should not be relatives or friends of the patient. The stigma of mental illness may prevent the openness needed during the encounter.

Select the example of a tort. a. The plan of care for a patient is not completed within 24 hours of the patient's admission. b. A nurse gives a prn dose of an antipsychotic drug to an agitated patient because the unit is short-staffed. c. An advanced practice nurse recommends hospitalization for a patient who is dangerous to self and others. d. A patient's admission status changed from involuntary to voluntary after the patient's hallucinations subside.

B A tort is a civil wrong against a person that violates his or her rights. Giving unnecessary medication for the convenience of staff controls behavior in a manner similar to secluding a patient-- thus, false imprisonment is a possible charge.

Which action by a nurse constitutes a breach of a patient's right to privacy? a. Documenting the patient's daily behavior during hospitalization b. Releasing information to the patient's employer without consent c. Discussing the patient's history with other staff during care planning d. Asking family to share information about a patient's pre-hospitalization behavior

B Release of information without patient authorization violates the patient's right to privacy. The other options are acceptable nursing practices. See relationship to audience response question.

An adolescent hospitalized after a violent physical outburst tells the nurse, "I'm going to kill my father, but you can't tell anyone." Select the nurse's best response. a. "You are right. Federal law requires me to keep clinical information private." b. "I am obligated to share that information with the treatment team." c. "Those kinds of thoughts will make your hospitalization longer." d. "You should share this thought with your psychiatrist."

B Breach of nurse-patient confidentiality does not pose a legal dilemma for nurses in these circumstances because a team approach to delivery of psychiatric care presumes communication of patient information to other staff members to develop treatment plans and outcome criteria. The patient should also know that the team has a duty to warn the father of the risk for harm.

Which nursing intervention demonstrates false imprisonment? a. A confused and combative patient says, "I'm getting out of here, and no one can stop me." The nurse restrains this patient without a health care provider's order and then promptly obtains an order. b. A patient has been irritating and attention seeking much of the day. A nurse escorts the patient down the hall saying, "Stay in your room, or you'll be put in seclusion." c. An involuntarily hospitalized patient with suicidal ideation runs out of the psychiatric unit. The nurse rushes after the patient and convinces the patient to return to the unit. d. An involuntarily hospitalized patient with homicidal ideation attempts to leave the facility. A nurse calls the security team and uses established protocols to prevent the patient from leaving.

B False imprisonment involves holding a competent person against his or her will. Actual force is not a requirement of false imprisonment. The individual needs only to be placed in fear of imprisonment by someone who has the ability to carry out the threat. If a patient is not competent (confused), then the nurse should act with beneficence. Patients admitted involuntarily should not be allowed to leave without permission of the treatment team.

A black patient says to a white nurse, "There's no sense talking about how I feel. You wouldn't understand because you live in a white world." The nurse's best action would be to a. explain, "Yes, I do understand. Everyone goes through the same experiences." b. say, "Please give an example of something you think I wouldn't understand." c. reassure the patient that nurses interact with people from all cultures. d. change the subject to one that is less emotionally disturbing.

B Having the patient speak in specifics rather than globally will help the nurse understand the patient's perspective. This approach will help the nurse engage the patient. Reassurance and changing the subject are not therapeutic techniques.

A patient diagnosed with schizophrenia tells the nurse, "The Central Intelligence Agency is monitoring us through the fluorescent lights in this room. The CIA is everywhere, so be careful what you say." Which response by the nurse is most therapeutic? a. "Let's talk about something other than the CIA." b. "It sounds like you're concerned about your privacy." c. "The CIA is prohibited from operating in health care facilities." d. "You have lost touch with reality, which is a symptom of your illness."

B It is important not to challenge the patient's beliefs, even if they are unrealistic. Challenging undermines the patient's trust in the nurse. The nurse should try to understand the underlying feelings or thoughts the patient's message conveys. The correct response uses the therapeutic technique of reflection. The other comments are nontherapeutic. Asking to talk about something other than the concern at hand is changing the subject. Saying that the CIA is prohibited from operating in health care facilities gives false reassurance. Stating that the patient has lost touch with reality is truthful, but uncompassionate.

A patient diagnosed with schizophrenia believes a local minister stirred evil spirits. The patient threatens to bomb a local church. The psychiatrist notifies the minister. Select the answer with the correct rationale. The psychiatrist a. released information without proper authorization. b. demonstrated the duty to warn and protect. c. violated the patient's confidentiality. d. avoided charges of malpractice.

B It is the health care professional's duty to warn or notify an intended victim after a threat of harm has been made. Informing a potential victim of a threat is a legal responsibility of the health care professional. It is not a violation of confidentiality.

A nurse prepares to administer a scheduled intramuscular injection of an antipsychotic medication to an outpatient diagnosed with schizophrenia. As the nurse swabs the site, the patient shouts, "Stop! I don't want to take that medicine anymore. I hate the side effects." Select the nurse's best action. a. Assemble other staff for a show of force and proceed with the injection, using restraint if necessary. b. Stop the medication administration procedure and say to the patient, "Tell me more about the side effects you've been having." c. Proceed with the injection but explain to the patient that there are medications that will help reduce the unpleasant side effects. d. Say to the patient, "Since I've already drawn the medication in the syringe, I'm required to give it, but let's talk to the doctor about delaying next month's dose."

B Patients diagnosed with mental illness retain their civil rights unless there is clear, cogent, and convincing evidence of dangerousness. The patient in this situation presents no evidence of dangerousness. The nurse, as an advocate and educator, should seek more information about the patient's decision and not force the medication.

Two hospitalized patients fight whenever they are together. During a team meeting, a nurse asserts that safety is of paramount importance, so treatment plans should call for both patients to be secluded to keep them from injuring each other. This assertion a. reinforces the autonomy of the two patients. b. violates the civil rights of both patients. c. represents the intentional tort of battery. d. correctly places emphasis on safety.

B Patients have a right to treatment in the least restrictive setting. Safety is important, but less restrictive measures should be tried first. Unnecessary seclusion may result in a charge of false imprisonment. Seclusion violates the patient's autonomy. The principle by which the nurse is motivated is beneficence, not justice. The tort represented is false imprisonment.

A patient experiencing psychosis asks a psychiatric technician, "What's the matter with me?" The technician replies, "Nothing is wrong with you. You just need to use some self-control." The nurse who overheard the exchange should take action based on a. the technician's unauthorized disclosure of confidential clinical information. b. violation of the patient's right to be treated with dignity and respect. c. the nurse's obligation to report caregiver negligence. d. the patient's right to social interaction.

B Patients have the right to be treated with dignity and respect. The technician's comment disregards the seriousness of the patient's illness. The Code of Ethics for Nurses requires intervention. Patient emotional abuse has been demonstrated, not negligence. An interaction with the technician is not an aspect of social interaction.

Which intervention best demonstrates that a nurse correctly understands the cultural needs of a hospitalized Asian American patient diagnosed with a mental illness? a. Encouraging the family to attend community support groups b. Involving the patient's family to assist with activities of daily living c. Providing educational pamphlets to explain the patient's mental illness d. Restricting homemade herbal remedies the family brings to the hospital

B The Asian community values the family in caring for each other. The Asian community uses traditional medicines and healers, including herbs for mental symptoms. The Asian community describes illness in somatic terms. The Asian community attaches a stigma to mental illness, so interfacing with the community would not be appealing.

A nurse cares for a first-generation American whose family emigrated from Germany. Which worldview about the source of knowledge would this patient likely have? a. Knowledge is acquired through use of affective or feeling senses. b. Science is the foundation of knowledge and proves something exists. c. Knowledge develops by striving for transcendence of the mind and body. d. Knowledge evolves from an individual's relationship with a supreme being.

B The European-American perspective of acquiring knowledge evolves from science. The distracters describe the beliefs of other cultural groups.

A nurse finds a psychiatric advance directive in the medical record of a patient currently experiencing psychosis. The directive was executed during a period when the patient was stable and competent. The nurse should a. review the directive with the patient to ensure it is current. b. ensure that the directive is respected in treatment planning. c. consider the directive only if there is a cardiac or respiratory arrest. d. encourage the patient to revise the directive in light of the current health problem.

B The nurse has an obligation to honor the right to self-determination. An advanced psychiatric directive supports that goal. Since the patient is currently psychotic, the terms of the directive now apply

An aide in a psychiatric hospital says to the nurse, "We don't have time every day to help each patient complete a menu selection. Let's tell dietary to prepare popular choices and send them to our unit." Select the nurse's best response. a. "Thanks for the suggestion, but that idea may not work because so many patients take MAOI (monoamine oxidase inhibitor) antidepressants." b. "Thanks for the idea, but it's important to treat patients as individuals. Giving choices is one way we can respect patients' individuality." c. "Thank you for the suggestion, but the patients' bill of rights requires us to allow patients to select their own diet." d. "Thank you. That is a very good idea. It will make meal preparation easier for the dietary department."

B The nurse's response to the aide should recognize patients' rights to be treated with dignity and respect as well as promote autonomy. This response also shows respect for the aide and fulfills the nurse's obligation to provide supervision of unlicensed personnel. The incorrect responses have flawed rationale or do not respect patients as individuals.

The patient says, "My marriage is just great. My spouse and I always agree." The nurse observes the patient's foot moving continuously as the patient twirls a shirt button. The conclusion the nurse can draw is that the patient's communication is a. clear. b. distorted. c. incongruous. d. inadequate.

B The patient's verbal and nonverbal communication in this scenario are incongruous. Incongruous messages involve transmission of conflicting messages by the speaker. The patient's verbal message that all was well in the relationship was modified by the nonverbal behaviors denoting anxiety. Data are not present to support the choice of the verbal message being clear, explicit, or inadequate.

During the first interview with a parent whose child died in a car accident, the nurse feels empathic and reaches out to take the patient's hand. Select the correct analysis of the nurse's behavior. a. It shows empathy and compassion. It will encourage the patient to continue to express feelings. b. The gesture is premature. The patient's cultural and individual interpretation of touch is unknown. c. The patient will perceive the gesture as intrusive and overstepping boundaries. d. The action is inappropriate. Psychiatric patients should not be touched.

B Touch has various cultural and individual interpretations. Nurses should refrain from using touch until an assessment is completed regarding the way in which the patient will perceive touch. The incorrect options present prematurely drawn conclusions.

A Native American patient describes a difficult childhood and dropping out of high school. The patient abused alcohol as a teenager to escape feelings of isolation but stopped 10 years ago. The patient now says, "I feel stupid. I've never had a good job. I don't help my people." Which nursing diagnosis applies? a. Risk for other-directed violence b. Chronic low self-esteem c. Deficient knowledge d. Social isolation

B The patient has given several indications of chronic low self-esteem.

A patient cries as the nurse explores the patient's feelings about the death of a close friend. The patient sobs, "I shouldn't be crying like this. It happened a long time ago." Which responses by the nurse facilitate communication? (Select all that apply.) a. "Why do you think you are so upset?" b. "I can see that you feel sad about this situation." c. "The loss of a close friend is very painful for you." d. "Crying is a way of expressing the hurt you are experiencing." e. "Let's talk about something else because this subject is upsetting you."

B, C, D Reflecting ("I can see that you feel sad," "This is very painful for you") and giving information ("Crying is a way of expressing hurt") are therapeutic techniques. "Why" questions often imply criticism or seem intrusive or judgmental. They are difficult to answer. Changing the subject is a barrier to communication.

A voluntarily hospitalized patient tells the nurse, "Get me the forms for discharge. I want to leave now." Select the nurse's best response. a. "I will get the forms for you right now and bring them to your room." b. "Since you signed your consent for treatment, you may leave if you desire." c. "I will get them for you, but let's talk about your decision to leave treatment." d. "I cannot give you those forms without your health care provider's permission."

C A voluntarily admitted patient has the right to demand and obtain release in most states. However, as a patient advocate, the nurse is responsible for weighing factors related to the patient's wishes and best interests. By asking for information, the nurse may be able to help the patient reconsider the decision. Facilitating discharge without consent is not in the patient's best interests before exploring the reason for the request.

A patient discloses several concerns and associated feelings. If the nurse wants to seek clarification, which comment would be appropriate? a. "What are the common elements here?" b. "Tell me again about your experiences." c. "Am I correct in understanding that." d. "Tell me everything from the beginning."

C Asking, "Am I correct in understanding that ..." permits clarification to ensure that both the nurse and patient share mutual understanding of the communication. Asking about common elements encourages comparison rather than clarification. The remaining responses are implied questions that suggest the nurse was not listening.

A patient tells the nurse, "I don't think I'll ever get out of here." Select the nurse's most therapeutic response. a. "Don't talk that way. Of course you will leave here!" b. "Keep up the good work, and you certainly will." c. "You don't think you're making progress?" d. "Everyone feels that way sometimes."

C By asking if the patient does not believe that progress has been made, the nurse is reflecting or paraphrasing by putting into words what the patient is hinting. By making communication more explicit, issues are easier to identify and resolve. The remaining options are nontherapeutic techniques. Telling the patient not to "talk that way" is disapproving. Saying that everyone feels that way at times minimizes feelings. Telling the patient that good work will always result in success is falsely reassuring.

A Mexican American patient puts a picture of the Virgin Mary on the bedside table. What is the nurse's best action? a. Move the picture so it is beside a window. b. Send the picture to the business office safe. c. Leave the picture where the patient placed it. d. Send the picture home with the patient's family.

C Cultural heritage is expressed through language, works of art, music, dance, customs, traditions, diet, and expressions of spirituality. This patient's prominent placement of the picture is an example of expression of cultural heritage and spirituality. The nurse should not move it unless the patient's safety is jeopardized.

A group activity on an inpatient psychiatric unit is scheduled to begin at 1000. A patient, who was recently discharged from U.S. Marine Corps, arrives at 0945. Which analysis best explains this behavior? a. The patient wants to lead the group and give directions to others. b. The patient wants to secure a chair that will be close to the group leader. c. The military culture values timeliness. The patient does not want to be late. d. The behavior indicates feelings of self-importance that the patient wants others to appreciate.

C Culture is more than ethnicity and social norms. it includes religious, geographic, socioeconomic, occupational, ability- or disability-related, and sexual orientation-related beliefs and behaviors. In this instance, the patient's military experience represents an aspect of the patient's behavior. The military culture values timeliness. The distracters represent misinterpretation of the patient's behavior and have no bearing on the situation.

After leaving work, a nurse realizes documentation of administration of a prn medication was omitted. This off-duty nurse phones the nurse on duty and says, "Please document administration of the medication for me. My password is alpha1." The nurse receiving the call should a. fulfill the request promptly. b. document the caller's password. c. refer the matter to the charge nurse to resolve. d. report the request to the patient's health care provider.

C Fraudulent documentation may be grounds for discipline by the state board of nursing. Referring the matter to the charge nurse will allow observance of hospital policy while ensuring that documentation occurs. Notifying the health care provider would be unnecessary when the charge nurse can resolve the problem. Nurses should not provide passwords to others.

A Haitian patient diagnosed with major depressive disorder tells the nurse, "There's nothing you can do. This is a punishment. The only thing I can do is see a healer." The culturally aware nurse assesses that the patient a. has delusions of persecution. b. has likely been misdiagnosed with depression. c. may believe the distress is the result of a curse or spell. d. feels hopeless and helpless related to an unidentified cause.

C Individuals of African American or Caribbean cultures who have a fatalistic attitude about illness may believe they are being punished for wrongdoing or are victims of witchcraft or voodoo. They may be reticent to share information about curses with therapists. No data are present in the scenario to support delusions. Misdiagnosis more often labels a patient with depression as having schizophrenia.

Which individual diagnosed with a mental illness may need involuntary hospitalization? An individual a. who has a panic attack after her child gets lost in a shopping mall. b. with visions of demons emerging from cemetery plots throughout the community. c. who takes 38 acetaminophen tablets after the person's stock portfolio becomes worthless. d. diagnosed with major depression who stops taking prescribed antidepressant medication.

C Involuntary hospitalization protects patients who are dangerous to themselves or others and cannot care for their own basic needs. Involuntary hospitalization also protects other individuals in society. An overdose of acetaminophen indicates dangerousness to self. The behaviors described in the other options are not sufficient to require involuntary hospitalization.

Which communication strategy would be most effective for a nurse to use during an assessment interview with an adult Native American patient? a. Open and friendly, ask direct questions, touch the patient's arm or hand occasionally for reassurance. b. Frequent nonverbal behaviors, such as gestures and smiles, make an unemotional face to express negatives. c. Soft voice, break eye contact occasionally, general leads and reflective techniques. d. Stern voice, unbroken eye contact, minimal gestures, direct questions.

C Native American culture stresses living in harmony with nature. Cooperative, sharing styles rather than competitive or intrusive approaches are preferred.

Which principle should guide the nurse in determining the extent of silence to use during patient interview sessions? a. A nurse is responsible for breaking silences. b. Patients withdraw if silences are prolonged. c. Silence can provide meaningful moments for reflection. d. Silence helps patients know that what they said was understood.

C Silence can be helpful to both participants by giving each an opportunity to contemplate what has transpired, weigh alternatives, and formulate ideas. A nurse breaking silences is not a principle related to silences. It is inaccurate to say that patients withdraw during long silences or that silence helps patients know that they are understood. Feedback helps patients know they have been understood.

A patient who has been hospitalized for 3 days with a serious mental illness says, "I've got to get out of here and back to my job. I get 60 to 80 messages a day, and I'm getting behind on my email correspondence." What is this patient's perspective about health and illness? a. Fateful, magical b. Eastern, holistic c. Western, biomedical d. Harmonious, religious

C The Western biomedical perspective holds the belief that sick people should be as independent and self-reliant as possible. Self-care is encouraged.

A new antidepressant is prescribed for an elderly patient diagnosed with major depressive disorder, but the dose is more than the usual geriatric dose. The nurse should a. consult a reliable drug reference. b. teach the patient about possible side effects and adverse effects. c. withhold the medication and confer with the health care provider. d. encourage the patient to increase oral fluids to reduce drug concentration.

C The dose of antidepressants for elderly patients is often less than the usual adult dose. The nurse should withhold the medication and consult the health care provider who wrote the order. The nurse's duty is to practice according to professional standards as well as intervene and protect the patient.

A nurse prepares to assess a newly hospitalized patient who moved to the United States 6 months ago from Somalia. The nurse should first determine a. if the patient's immunizations are current. b. the patient's religious preferences. c. the patient's specific ethnic group. d. whether an interpreter is needed.

D The assessment depends on communication. The nurse should first determine whether an interpreter is needed. The other information can be subsequently assessed.

Documentation in a patient's chart shows, "Throughout a 5-minute interaction, patient fidgeted and tapped left foot, periodically covered face with hands, and looked under chair while stating, 'I enjoy spending time with you.'" Which analysis is most accurate? a. The patient is giving positive feedback about the nurse's communication techniques. b. The nurse is viewing the patient's behavior through a cultural filter. c. The patient's verbal and nonverbal messages are incongruent. d. The patient is demonstrating psychotic behaviors.

C When a verbal message is not reinforced with nonverbal behavior, the message is confusing and incongruent. It is inaccurate to say that the patient is giving positive feedback about the nurse's communication techniques. The concept of a cultural filter is not relevant to the situation because a cultural filter determines what we will pay attention to and what we will ignore. Data are insufficient to draw the conclusion that the patient is demonstrating psychotic behaviors.

A Vietnamese patient's family reports that the patient has wind illness. Which menu selection will be most helpful for this patient? a. Iced tea b. Ice cream c. Warm broth d. Gelatin dessert

C Wind illness is a culture-bound syndrome found in the Chinese and Vietnamese population. It is characterized by a fear of cold, wind, or drafts. It is treated by keeping very warm and avoiding foods, drinks, and herbs that are cold. Warm broth would be most in sync with the patient's culture and provide the most comfort. The distracters are cold foods.

In which situations would a nurse have the duty to intervene and report? (Select all that apply.) a. A peer has difficulty writing measurable outcomes. b. A health care provider gives a telephone order for medication. c. A peer tries to provide patient care in an alcohol-impaired state. d. A team member violates relationship boundaries with a patient. e. A patient refuses medication prescribed by a licensed health care provider.

C, D Both keyed answers are events that jeopardize patient safety. The distracters describe situations that may be resolved with education or that are acceptable practices.

Which comments by a nurse demonstrate use of therapeutic communication techniques? (Select all that apply.) a. "Why do you think these events have happened to you?" b. "There are people with problems much worse than yours." c. "I'm glad you were able to tell me how you felt about your loss." d. "I noticed your hands trembling when you told me about your accident." e. "You look very nice today. I'm proud you took more time with your appearance."

C, D The correct responses demonstrate use of the therapeutic techniques making an observation and showing empathy. The incorrect responses demonstrate minimizing feelings, probing, and giving approval, which are nontherapeutic techniques.

Why is the study of culture so important for psychiatric nurses in the United States? (Select all that apply.) a. Psychiatric nurses often practice in other countries. b. Psychiatric nurses must advocate for the traditions of the Western culture. c. Cultural competence helps protect patients from prejudice and discrimination. d. Patients should receive information about their illness and treatment in terms they understand. e. Psychiatric nurses often interface with patients and their significant others over a long period of time.

C, D, E One purpose of cultural competence is for the psychiatric nurse to relate and explain information about the patient's illness and treatment in an understandable way, incorporating the patient's own beliefs and values. A fundamental aspect of nursing practice is advocacy. Cultural competence promotes recognition of prejudices in care, such as stigma and misdiagnosis. Psychiatric nurses often interface with patients and families over years and in community settings.

Which actions violate the civil rights of a psychiatric patient? The nurse (Select all that apply) a. performs mouth checks after overhearing a patient say, "I've been spitting out my medication." b. begins suicide precautions before a patient is assessed by the health care provider. c. opens and reads a letter a patient left at the nurse's station to be mailed. d. places a patient's expensive watch in the hospital business office safe. e. restrains a patient who uses profanity when speaking to the nurse.

C, E The patient has the right to send and receive mail without interference. Restraint is not indicated because a patient uses profanity

While performing a mental status examination on a client, the nurse notices that the client's facial expression constantly appears angry. This information should be recorded as part of the client's C. affect.

C. affect Description of the client's facial expression is described as affect. Facial expression is not described in the areas of behavior, appearance, or thought process.

A nurse interacts with a newly hospitalized patient. Select the nurse's comment that applies the communication technique of "offering self." a. "I've also had traumatic life experiences. Maybe it would help if I told you about them." b. "Why do you think you had so much difficulty adjusting to this change in your life?" c. "I hope you will feel better after getting accustomed to how this unit operates." d. "I'd like to sit with you for a while to help you get comfortable talking to me."

D "Offering self" is a technique that should be used in the orientation phase of the nurse-patient relationship. Sitting with the patient, an example of "offering self," helps to build trust and convey that the nurse cares about the patient. Two incorrect responses are ineffective and nontherapeutic. The other incorrect response is therapeutic but is an example of "offering hope."

A newly admitted acutely psychotic patient is a private patient of the medical director and a private-pay patient. To whom does the psychiatric nurse assigned to the patient owe the duty of care? a. Medical director b. Hospital c. Profession d. Patient

D Although the nurse is accountable to the health care provider, the agency, the patient, and the profession, the duty of care is owed to the patient. This duty reflects both legal and ethical standards of nursing practice.

What is the legal significance of a nurse's action when a patient verbally refuses medication and the nurse gives the medication over the patient's objection? The nurse a. has been negligent. b. committed malpractice. c. fulfilled the standard of care. d. can be charged with battery.

D Battery is an intentional tort in which one individual violates the rights of another through touching without consent. Forcing a patient to take medication after the medication was refused constitutes battery. The charge of battery can be brought against the nurse. The medication may not necessarily harm the patient-- harm is a component of malpractice.

A nurse begins work in an agency that provides care to members of a minority ethnic population. The nurse will be better able to demonstrate cultural competence after a. identifying culture-bound issues. b. implementing scientifically proven interventions. c. correcting inferior health practices of the population. d. exploring commonly held beliefs and values of the population.

D Cultural competence is dependent on understanding the beliefs and values of members of a different culture. A nurse who works with an individual or group of a culture different from his or her own must be open to learning about the culture. The other options have little to do with cultural competence or represent only a portion of the answer.

Which Western cultural feature may result in establishing unrealistic outcomes for patients of other cultural groups? a. Interdependence b. Present orientation c. Flexible perception of time d. Direct confrontation to solve problems

D Directly confronting problems is a highly valued approach in the American culture but not part of many other cultures in which harmony and restraint are valued

A patient experiencing psychosis became aggressive, struck another patient, and required seclusion. Select the best documentation. a. Patient struck another patient who attempted to leave day room to go to bathroom. Seclusion necessary at 1415. Plan: Maintain seclusion for 8 hours and keep these two patients away from each other for 24 hours. b. Seclusion ordered by physician at 1415 after command hallucinations told the patient to hit another patient. Careful monitoring of patient maintained during period of seclusion. c. Seclusion ordered by MD for aggressive behavior. Begun at 1415. Maintained for 2 hours without incident. Outcome: Patient calmer and apologized for outburst. d. Patient pacing, shouting. Haloperidol 5 mg given PO at 1300. No effect by 1315. At 1415 patient yelled, "I'll punch anyone who gets near me," and struck another patient with fist. Physically placed in seclusion at 1420. Seclusion order obtained from MD at 1430.

D Documentation must be specific and detail the key aspects of care. It should demonstrate implementation of the least restrictive alternative. Justification for why a patient was secluded should be recorded, along with interventions attempted in an effort to avoid seclusion. Documentation should include a description of behavior and verbalizations, interventions tried and their outcomes, and the name of the health care provider ordering the use of seclusion.

Which patient meets criteria for involuntary hospitalization for psychiatric treatment? The patient who a. is noncompliant with the treatment regimen. b. fraudulently files for bankruptcy. c. sold and distributed illegal drugs. d. threatens to harm self and others.

D Involuntary hospitalization protects patients who are dangerous to themselves or others and cannot care for their own basic needs. Involuntary commitment also protects other individuals in society. The behaviors described in the other options are not sufficient to require involuntary hospitalization

A Puerto Rican American patient uses dramatic body language when describing emotional discomfort. Which analysis most likely explains the patient's behavior? The patient a. has a histrionic personality disorder. b. believes dramatic body language is sexually appealing. c. wishes to impress staff with the degree of emotional pain. d. belongs to a culture in which dramatic body language is the norm.

D Members of Hispanic American subcultures tend to use high affect and dramatic body language as they communicate. The other options are more remote possibilities.

A Native American patient sadly describes a difficult childhood. The patient abused alcohol as a teenager but stopped 10 years ago. The patient now says, "I feel stupid and good for nothing. I don't help my people." How should the treatment team focus planning for this patient? a. Psychopharmacological and somatic therapies should be central techniques. b. Apply a psychoanalytical approach, focused on childhood trauma. c. Depression and alcohol abuse should be treated concurrently. d. Use a holistic approach, including mind, body, and spirit.

D Native Americans, because of their beliefs in the interrelatedness of parts and about being in harmony with nature, respond best to a holistic approach.

A patient in the emergency department shows a variety of psychiatrical symptoms, including restlessness and anxiety. The patient says, "I feel sad because evil spirits have overtaken my mind." Which worldview is most applicable to this individual? a. Eastern/balance b. Southern/holistic c. Western/scientific d. Indigenous/harmony

D Persons of an indigenous worldview believe disease results from a lack of personal, interpersonal, environmental, or spiritual harmony and that evil spirits exist. The holism of body-mind-spirit is a key component of this view. If one believes an evil spirit has taken control, distress results. Western and Eastern worldviews do not embrace spirits.

A black patient, originally from Haiti, has a diagnosis of major depressive disorder. A colleague tells the nurse, "This patient often looks down and is reluctant to share feelings. However, I've observed the patient spontaneously interacting with other black patients." Select the nurse's best response. a. "Black patients depend on the church for support. Have you consulted the patient's pastor?" b. "Encourage the patient to talk in a group setting. It will be less intimidating than one-to-one interaction." c. "Don't take it personally. Black patients often have a resentful attitude that takes a long time to overcome." d. "The patient may have difficulty communicating in English. Have you considered using a cultural broker?"

D Society expects a culturally diverse patient to accommodate and use English. Feelings are abstract, which requires a greater command of the language. This may be especially difficult during episodes of high stress or mental illness. Cultural brokers can be helpful with language and helping the nurse to understand the Haitian worldview and cultural nuances.

A Filipino American patient had a nursing diagnosis of situational low self-esteem related to poor social skills as evidenced by lack of eye contact. Interventions were applied to increase the patient's self-esteem but after 3 weeks, the patient's eye contact did not improve. What is the most accurate analysis of this scenario? a. The patient's eye contact should have been directly addressed by role playing to increase comfort with eye contact. b. The nurse should not have independently embarked on assessment, diagnosis, and planning for this patient. c. The patient's poor eye contact is indicative of anger and hostility that were unaddressed. d. The nurse should have assessed the patient's culture before making this diagnosis and plan.

D The amount of eye contact a person engages in is often culturally determined. In some cultures, eye contact is considered insolent, whereas in others eye contact is expected. Asian Americans, including persons from the Philippines, often prefer not to engage in direct eye contact.

A school age child tells the school nurse, "Other kids call me mean names and will not sit with me at lunch. Nobody likes me." Select the nurse's most therapeutic response. a. "Just ignore them and they will leave you alone." b. "You should make friends with other children." c. "Call them names if they do that to you." d. "Tell me more about how you feel."

D The correct response uses exploring, a therapeutic technique. The distracters give advice, a nontherapeutic technique.

Which documentation of a patient's behavior best demonstrates a nurse's observations? a. Isolates self from others. Frequently fell asleep during group. Vital signs stable. b. Calmer-- more cooperative. Participated actively in group. No evidence of psychotic thinking. c. Appeared to hallucinate. Frequently increased volume on television, causing conflict with others. d. Wore four layers of clothing. States, "I need protection from evil bacteria trying to pierce my skin."

D The documentation states specific observations of the patient's appearance and the exact statements made. The other options are vague or subjective statements and can be interpreted in different ways.

A family member of a patient with delusions of persecution asks the nurse, "Are there any circumstances under which the treatment team is justified in violating a patient's right to confidentiality?" The nurse should reply that confidentiality may be breached a. under no circumstances. b. at the discretion of the psychiatrist. c. when questions are asked by law enforcement. d. if the patient threatens the life of another person.

D The duty to warn a person whose life has been threatened by a psychiatric patient overrides the patient's right to confidentiality. The right to confidentiality is not suspended at the discretion of the therapist or for legal investigations.

In a team meeting a nurse says, "I'm concerned about whether we are behaving ethically by using restraint to prevent one patient from self-mutilation, while the care plan for another self-mutilating patient requires one-on-one supervision." Which ethical principle most clearly applies to this situation? a. Beneficence b. Autonomy c. Fidelity d. Justice

D The nurse is concerned about justice, that is, fair distribution of care, which includes treatment with the least restrictive methods for both patients. Beneficence means promoting the good of others. Autonomy is the right to make one's own decisions. Fidelity is the observance of loyalty and commitment to the patient.

A patient says to the nurse, "I dreamed I was stoned. When I woke up, I felt emotionally drained, as though I hadn't rested well." Which response should the nurse use to clarify the patient's comment? a. "It sounds as though you were uncomfortable with the content of your dream." b. "I understand what you're saying. Bad dreams leave me feeling tired, too." c. "So you feel as though you did not get enough quality sleep last night?" d. "Can you give me an example of what you mean by 'stoned'?"

D The technique of clarification is therapeutic and helps the nurse examine the meaning of the patient's statement. Asking for a definition of "stoned" directly asks for clarification. Restating that the patient is uncomfortable with the dream's content is parroting, a nontherapeutic technique. The other responses fail to clarify the meaning of the patient's comment.

A white patient of German descent rocks back and forth, grimaces, and rubs both temples. What is the nurse's best action? a. Assess the patient for extrapyramidal symptoms. b. Sit beside the patient and rock in sync. c. Offer to pray with the patient. d. Assess the patient for pain.

D This patient of German descent would hold a Western worldview and be stoic about pain. This patient will keep pain as silent as possible and be reluctant to disclose pain unless the nurse actively assesses for it.

Which individual diagnosed with mental illness may need emergency or involuntary admission? The individual who a. resumes using heroin while still taking naltrexone (ReVia). b. reports hearing angels playing harps during thunderstorms. c. does not keep an outpatient appointment with the mental health nurse. d. throws a heavy plate at a waiter at the direction of command hallucinations.

D Throwing a heavy plate is likely to harm the waiter and is evidence of dangerousness to others. This behavior meets the criteria for emergency or involuntary hospitalization for mental illness. The behaviors in the other options evidence mental illness but not dangerousness. See related audience response question.

During an interview, a patient attempts to shift the focus from self to the nurse by asking personal questions. The nurse should respond by saying: a. "Why do you keep asking about me?" b. "Nurses direct the interviews with patients." c. "Do not ask questions about my personal life." d. "The time we spend together is to discuss your concerns."

D When a patient tries to focus on the nurse, the nurse should refocus the discussion back onto the patient. Telling the patient that interview time should be used to discuss patient concerns refocuses discussion in a neutral way. Telling patients not to ask about the nurse's personal life shows indignation. Saying that nurses prefer to direct the interview reflects superiority. "Why" questions are probing and nontherapeutic.

A nurse is caring for a client diagnosed with paranoid schizophrenia, asthma, generalized anxiety disorder, and borderline personality disorder. Which of the following diagnoses should the nurse expect to find included in Axis II of this client's DSM-IV-TR axis diagnosis? D. Borderline personality disorder

D. Borderline personality disorder Personality disorders and mental retardation are included in Axis II. Paranoid schizophrenia and generalized anxiety disorder are found in Axis I. Asthma and other general medical conditions are found in Axis III.

Which criterion is NOT essential when the nurse plans nursing interventions designed to meet a specific goal? Economic

Economical Correct Although expense should be considered, interventions are chosen based on the other options and not on their economic value.

Which activity is NOT considered a purpose of the initial psychiatric assessment? Evaluating the results of intervention

Evaluating the results of intervention Correct At an initial assessment, no interventions would have taken place; hence evaluation is not a purpose of the initial contact.

Which nursing diagnosis for a psychiatric client is correctly structured and worded? Imbalanced nutrition: less than body requirements related to poor self-concept as evidenced by reporting "I'm not worthy of eating"

Imbalanced nutrition: less than body requirements related to poor self-concept as evidenced by reporting "I'm not worthy of eating" Correct This diagnosis contains all the required components: problem statement, the etiology, and supporting data. REF: Page 123

The mental status examination aids in the collection of what type of data? Objective

Objective Correct The mental status exam mostly aids in the collection of objective data.

A tool the novice nurse might refer to when writing treatment results criteria is th Nursing Outcomes Classification (NOC).

Nursing Outcomes Classification (NOC). Correct The Nursing Outcomes Classification is a publication used as a resource across the United States.

What three structural components comprise a nursing diagnosis? Problem, etiology, supporting data

Problem, etiology, supporting data Correct

Joel is a 43-year-old patient being seen in the mental health clinic with depression. Joel states, "I have always been a practicing Jew, but in the past few months I am questioning everything. I just don't know if I believe in it anymore." Which of the following nursing diagnoses best describes Joel's comment? Spiritual distress

Spiritual distress Correct Joel is expressing distress regarding his religion and spiritual well-being. Joel could be experiencing ineffective coping, but this does not directly relate to his comment. There is nothing in Joel's comment that would lead to the conclusion the patient is having thoughts of harming himself. Joel's comment does not describe hopelessness.

Which of the following are examples of subjective assessment data? (Select all that apply.) ● ● "Client states he has no reason to live." ● ● "Client meditates for relaxation." ● ● "Client states that he drinks three beers a day."

Subjective data includes psychosocial information about the client's thoughts, actions and feelings that can only be described by the client. Objective data is based on observable or verifiable facts.

You are conducting an admission interview with Callie, who was raped 2 weeks ago. When you ask Callie about the rape, she becomes very anxious and upset and begins to sob. Your best course of actions would be to: acknowledge that the topic of the rape is upsetting to Callie and reassure her that it can be discussed at another time when she feels more comfortable.

acknowledge that the topic of the rape is upsetting to Callie and reassure her that it can be discussed at another time when she feels more comfortable. Correct The best atmosphere for conducting an assessment is one with minimal anxiety on the patient's part. If a topic causes distress, it is best to abandon the topic at that time. It is important not to pry or push for information that is difficult for the patient to discuss. The use of silence continues to expect the patient to discuss the topic now. Reassurance of confidentiality continues to expect the patient to discuss the topic now.

In psychiatric nursing, assessment of a "client" refers exclusively to an individual, family, group, or community.

an individual, family, group, or community. Correct Standards of practice for psychiatric nursing indicate that the client can be an individual, a family, a group, or a community.

A nurse is about to interview a client whose glasses and hearing aid were placed in safe-keeping when she was admitted. Before beginning the interview, the nursing intervention that will best facilitate data collection is to assist the client in putting on glasses and hearing aid.

assist the client in putting on glasses and hearing aid. Correct A client whose hearing or sight is impaired may have difficulty providing information if these items have been removed from his or her possession. Assisting the client in wearing these assistive devices is the best initial intervention. REF: Page 118-119

The primary source for data collection during a psychiatric nursing assessment is the client's own words and actions.

client's own words and actions. Correct The client should always be considered the primary data source. At times, however, the client will be unable to fulfill this role

During the initial assessment interview with a psychiatric client, the nurse should regard the spiritual assessment as important to complete.

important to complete. Correct For many clients, religious or spiritual practices are an important part of the quality of their lives. Nurses should support the spiritual dimension of the person. To do so, assessment is necessary.

High levels of anxiety and maladaptive behavior are seen in all areas in the health care setting.

in all areas in the health care setting. Correct Anxiety occurs whenever individuals are faced with unfamiliar circumstances or other threats to the self. The health care setting presents many possible threats to the self, such as illness, disability, surgery, and pain.

the principle that is the basis of nursing outcome planning is individuals have the right to autonomy to make decisions that affect them.

individuals have the right to autonomy to make decisions that affect them. Correct This is the only true statement. The nurse and the client should work collaboratively because each has knowledge to contribute to planning for the attainment of mutually derived outcomes. REF: Page 124-125

The client's priority nursing diagnosis has been established as risk for self-directed violence: suicide related to multiple losses. The priority outcome would be that the client will refrain from attempting suicide.

refrain from attempting suicide. Correct Refraining from suicidal attempts is the only outcome that addresses the risk for self-directed violence. The absence of a feeling of powerlessness is not appropriate for the stated nursing diagnosis. The remaining options are interventions. REF: Page 124-125

Interviewer anxiety during an assessment interview is most likely to be a result of the client's perception of the interviewer's ability to help.

the client's perception of the interviewer's ability to help. Correct Whenever a client is in doubt about the helpfulness of the interviewer, anxiety is generated. The interviewer can "tune in" to the client's anxiety by empathy. REF: Page 116-117

The nurse best ensures appropriate client care when choosing an intervention from a Nursing Interventions Classification that matches both the defining data and the nursing diagnosis.

the defining data and the nursing diagnosis. Correct When choosing nursing interventions from the Nursing Interventions Classification or some other source, the nurse selects interventions that fit the nursing diagnosis (e.g., risk for suicide) and that match the defining data.

The most likely factor to interfere with data collection in an initial assessment interview of an older adult is whether the client has any physical deficiencies.

whether the client has any physical deficiencies. Correct While all the options can interfere, the most prevalent one affecting the data collected is any physical and/or cognitive deficiencies that client may possess. REF: Page 122-123


Set pelajaran terkait

Smart Book-Chapter 6 Cash, Fraud, and Internal Control

View Set

Financial Management Exam 1 Terms (chapters 1-4)

View Set

Unit 4 Q. MR Imaging Parameters (16 Q 32 MIn)

View Set

Causes of the Texas Revolution: Important Events

View Set